LSAT and Law School Admissions Forum

Get expert LSAT preparation and law school admissions advice from PowerScore Test Preparation.

User avatar
 Dave Killoran
PowerScore Staff
  • PowerScore Staff
  • Posts: 5853
  • Joined: Mar 25, 2011
|
#47021
Complete Question Explanation
(The complete setup for this game can be found here: lsat/viewtopic.php?t=15947)

The correct answer choice is (B)

The information in the question stem produces the following setup:
o97_game_#4_#21_diagram 1.png
Since R, Q, and U speak first or second at meetings 1 and 2, T and S must speak in the first two slots of meeting 3. Since T speaks fifth at meeting 2, and fourth at meeting 1, T must speak first at meeting 3, and it follows that S must speak second at meeting 3. Accordingly, answer choices (A), (C), and (D) can be eliminated. Answer choice (E) can also be eliminated since if U spoke fifth, U would have to speak first at another meeting and that is not possible in this situation. Thus, answer choice (B) is correct.
You do not have the required permissions to view the files attached to this post.

Get the most out of your LSAT Prep Plus subscription.

Analyze and track your performance with our Testing and Analytics Package.